Retirement Planning and Employee Benefits

Réussis tes devoirs et examens dès maintenant avec Quizwiz!

In 2021, Lisa and Kenny move from Texas to Georgia for Lisa to accept a new job. Her employer reimburses her $10,000 for the moving expenses of which $1,500 was for pre-move house hunting expenses. How much of the reimbursed moving expenses are taxable to Lisa?

$10,000 The full amount will be taxable to Lisa. Effective for years after 2017, moving expenses are not deductible by taxpayers (unless active duty military). 2017 TCJA

Tracy, age 46, is a self-employed financial planner and has Schedule C income from self-employment of $56,000. He has failed to save for retirement until now. Therefore, he would like to make the maximum contribution to his profit sharing plan. How much can he contribute to his profit sharing plan account?

$10,409 $56,000 Schedule C net income -3,956 (less ½ self-employment taxes at 15.3% × 0.9235) $52,044 Net self-employment income × 0.20 (0.25/1.25) $10,409 Keogh profit sharing contribution amount

Danielle has worked for the City of Buffalo for the last 20 years. She has deferred $19,500 into her 457(b) plan for 2021. She will attain her normal retirement age under the City's 457(b) plan in 2022. Danielle has prior unused deferral amount of $45,000 as of December 31, 2020. How much can Danielle contribute as her three-year catch-up contribution in 2021?

$19,500 Since the plan's normal retirement age for Danielle is 2022, Danielle would be allowed to defer an additional $19,500 in 2021. This is within the three years of the plan's normal retirement age and Danielle has sufficient prior unused deferral.

Jared, age 45, earns $300,000 per year and is a participant in his employer's 401(k) plan. Ignoring the ADP Test requirements, what is the maximum amount that Jared can defer under the 401(k) plan in 2021?

$19,500 The deferral limitation for 2021 is $19,500, and Jared could defer an additional $6,500 as a catch-up contribution if he was over 50.

Isse Peking is the manager of Airline Highway Motel. Isse lives in Unit 12. He was given the option to live at the motel if he would also look after the night auditing (the value of his reviews is $400 per month) responsibilities. The value of the motel unit on a monthly basis is $800, but Unit 12 rents on a daily basis for $100 per day. How much, if any, does Isse have to include in his gross income for living on the premises of his employer?

$800 per month Isse is not required by the employer to live on the premises and therefore must include the value of the lodging in his gross income.

Larry, age 55, is employed by BB Trucking Company as a tire repair specialist. He earns $62,000 per year. He received an allocation of $32,000 to his employer-provided profit sharing plan for the year. If BB Trucking does not match employee deferrals, what is the maximum amount Larry can defer to his 401(k) plan for the 2021 plan year?

$26,000 The general employee elective deferral limitation for 2021 is $19,500, and Jared can defer an additional $6,500 (2021) as a catch-up contribution because he is over 50.

Dorothy, age 45, earns $125,000 per year and has group life insurance through her employer for 2x her salary. If the Table I for IRC Section 79 cost is $.15, how much taxable income must Dorothy impute for the year?

$360 125,000 × 2 = 250,000 250,000 - 50,000 = 200,000 200,000/1,000 × .15 × 12 = $360

Taylor, age 65, retires from Tickle Tile corporation and receives 25,000 shares of Tickle Tile stock with a fair market value of $500,000. Taylor recognized $48,000 of ordinary income upon the distribution. What is Taylor's NUA immediately after the distribution?

$452,000 The NUA immediately after the distribution is $452,000 ($500,000 - $48,000). The FMV less the ordinary income recognized.

Hasani died December 31, 2020, at age 50 leaving his wife Jamille, age 49, and 4 children ages 4, 7, 15 and 17. His PIA is $2,500 per month. If Jamille goes back to work in 2021 and makes $100,000 how much approximately will she receive for the care of the children (ignoring family maximums)?

$5,625 Jamille will receive a benefit for each child she cares for under age 16. Each child under age 18 (19 if in secondary school - ie high school) will also receive a benefit directly. For the care of the children, Jamille will receive 75% of Hasani's calculated PIA. 2,500 x 75% = 1,875 for each of the three children under 16 = $5,625

Cavin sells stock several years after he received it as a distribution from a qualified stock bonus plan. When the stock was distributed, he had a net unrealized appreciation of $7,500. Cavin also had ordinary income from the distribution of $29,000. The fair market value of the stock and the sales price at the time of sale was $81,000. How much of the sale price will be subject to long-term capital gain treatment?

$52,000 Sale Price - Adjusted Basis. $81,000 - $29,000 = $52,000 long-term capital gain.

Jason turned 70 in November of 2020 and is retired. He was a participant in his employer's profit sharing plan. His profit sharing plan had an account balance of $250,000 on December 31 of this year, and $200,000 on December 31 of last year. According to the Uniform Lifetime Table the factors for ages 70, 71, and 72 are 27.4, 26.5, and 25.6 respectively. What is Jason's approximate required minimum distribution for 2021?

0 SECURE Act changed the Required Begin Date for Required Minimum Distributions to age 72 for anyone turning 70 1/2 after 12/31/19. Jason will take his first RMD for the tax year 2022.

Which of the following are characteristics of a phantom stock plan? 1. Benefits are paid in cash. 2. There is no equity dilution from additional shares being issued.

1 and 2 The employee does not actually receive stock in a phantom plan. Instead, the employee receives credits for the stock and the benefits are later paid in cash.

When calculating the Wage Replacement Ratio (WRR), what percentage of income is subtracted for a self-employed individual, under the Social Security wage base, for Social Security and Medicare Taxes excluding the Additional Medicare Tax?

15.30% This is an important point to stress as many clients are self-employed and pay both employer and employee portions of the tax, 6.2% social security, 1.45% medicare (7.65%) for the employee portion, and the same for the employer portion for a total of 15.3% (7.65 + 7.65).

What is the maximum number of employees that a company with a health plan can have and not be subject to the COBRA rules?

19 A company with less than 20 employees is not subject to COBRA even if they have a health plan.

Joe's full retirement age is 67. He is considering retiring early, at age 62. How much will his retirement benefit be reduced by, if he elects to receive social security retirement benefits at age 62?

30% 5/9 × 36 = 20% 5/12 × 24 = 10% 20% + 10% = 30%

Company A has been capitalized by MJBJ Vulture Capital, a venture capital company. Company A's cash flows are expected to fluctuate significantly from year to year, due to phenomenal growth. They expect to go public within three years. What is the best qualified plan for them to consider adopting?

A stock bonus plan will allow equity participation without the use of cash flows and the public offering will eventually provide liquidity.

The Investment Issues are:

-The Asset Classes invested in -The Inflation Rates -The Client's risk Tolerance

The Savings Issues are:

-The Savings Amount -The Savings Rate -The Timing of Savings

Which of the following vesting schedules may a top-heavy qualified profit sharing plan use? A. 1 to 5 year graduated B. 5 year cliff. C. 3 to 7 year graduated D. 4 to 8 year graduated.

A. As a result of the PPA 2006, qualified profit sharing plans must use a vesting schedule that provides participants with vested benefits at least as rapidly as either a 2 to 6 year graduated vesting schedule or a 3-year cliff vesting schedule. This requirement applies without regard to whether the profit sharing plan is a top-heavy plan. Options B, C, and D all vest less rapidly than the required schedule.

Which of the following entities is not permitted to establish a 401(k) plan? A. A Government entity. B. A LLC. C. A Partnership. D. A Tax-exempt entity.

A. A government entity can no longer establish a 401(k) plan. The remaining entities may establish a 401(k) plan.

Which of the following factors may affect a person's individual retirement planning? 1. Work life expectancy 2. Retirement life expectancy 3. Inflation 4. Savings rate

All of the above All of the options may affect a retirement plan either positively or negatively. Reduced work life expectancy may provide an insufficient savings period, while an increased retirement life expectancy increases capital needs for retirement. A low savings rate may create an inability to meet capital requirements. Increased inflation rates will reduce purchasing power.

Which of the following retirement capital needs analysis methods liquidates a lump amount to zero during retirement? A. Annuity Method B. Capital Preservation C. Purchasing Power Preservation D. None of the Above

Annuity Method

See Picture

Answer 1 - Yes >5% last year Answer 2 - Yes, >5% this year Answer 3 - Yes, >130,000 (2021), but based on last year's compensation.

The following statements concerning retirement plan service requirements for most qualified plans are correct EXCEPT: A. The term "year of service" refers to an employee who has worked at least 1,000 hours during the initial 12-month period after being employed. B. If an employee hired on October 5, 20X1 has worked at least 1,000 hours or more by October 4, 20X2, he has acquired a year of service the day after he worked his 1,000th hour. C. An employer has the option of increasing the one-year of service requirement to 2 years of service. D. Once an employee, who is over the age of 21, attains the service requirement of the plan, the employer cannot make the employee wait more than an additional six months to participate in the plan.

B. If an employee hired on October 5, 20X1 has worked at least 1,000 hours or more by October 4, 20X2, he has acquired a year of service the day after he worked his 1,000th hour. Option B is incorrect because the employee would NOT acquire a year of service the day after he worked his 1,000th hour, but after twelve months AND 1,000 hours.

Which of the following statements are reasons to delay eligibility of employees to participate in a retirement plan? 1. Employees don't start earning benefits until they become plan participants (except in defined benefit plans, which may count prior service). 2. Since turnover is generally highest for employees in their first few years of employment and for younger employees, it makes sense from an administrative standpoint to delay their eligibility.

Both 1 and 2

Which of the following capital needs analysis methods mitigates the risk of outliving retirement funds and assumes at life expectancy the same account balance as when the client enters retirement? A. Capital Preservation Model B. Present Value of an Annuity Due Model C. Purchasing Power Preservation Model D. Serial Accumulation Model

Capital Preservation Model The Capital Preservation Model assumes at life expectancy, as estimated in the annuity method, the client has exactly the same account balance as he/she started with at retirement. So if life expectancy is exceeded there is still capital available.

Financial Infidelity

Characterized by engaging in significant financial transactions without the knowledge and support of a spouse or partner. While many people engage in financial transactions of some sort without the knowledge of a partner, it becomes problematic when the transactions are significant in amount, such as hiding money, accumulating debt, gambling, or investing in illiquid assets that require the use of debt and leverage excessive preoccupation with shopping and spending that leads to distress. Can be a response to negative feelings and or events in one's life.

Compulsive Buying Disorder (CBD)

Characterized by excessive preoccupation with shopping and spending that leads to distress. Can be a response to negative feelings and or events in one's life..

Gambling

Characterized by the urge to gamble even when one's resources are low or even depleted. It is a recognized psychiatric condition often associated with emotional highs and lows or winning and losing a bet.

Wage Replacement Ratio (WRR) The Bottom-Up (Budgeting) Approach

Determines which preretirement expenses and expenditures will persist during the retirement years. Reviews each current expense and is more accurate for older individuals as the expenses are not as likely to change as much.

All of the following plans may integrate with Social Security EXCEPT: A. ESOP B. Profit Sharing Plan C. Defined Benefit D. Target Benefit

ESOP

Distributions from qualified plans are always treated as ordinary income. True or False?

False

Fixed-income securities generally provide the best long-term hedge against inflation and loss of purchasing power.

False

True or False: Social Security benefits are based on the average of the three highest paid years for a covered worker.

False

Which statements are correct regarding penalties associated with IRA accounts? Distributions made prior to 59½ are subject to the 10% premature distribution penalty. There is a 50% excise tax on a required minimum distribution not made by April 1 of the year following the year in which age 72 is attained.

I and II. Statements I and II are both correct. SECURE Act 2019 changed Required Minimum Distributions to age 72 as a start date. Participants will have until April 1 following the year they turn 72 to begin their RMDs.

One of the requirements for SIMPLE plans is that they can only be established for companies who employ 100 or fewer employees.

True

Company A has been capitalized by MJBJ Vulture Capital, a venture capital company. Company A's cash flows are expected to fluctuate significantly from year to year, due to phenomenal growth. They expect to go public within three years. Which of the following would be the best qualified plan for them to consider adopting? A. A profit sharing plan. B. A new comparability plan. C. A 401(k) plan with a match. D. A stock bonus plan.

The answer is D. A stock bonus plan will allow equity participation without the use of cash flows and the public offering will eventually provide liquidity.

The early distribution penalty of 10% does not apply to IRA distributions: I. Made after attainment of the age of 55 and separated from service. II. Made for the purpose of paying qualified higher education costs. III. Paid to a designated beneficiary after the death of the account owner who had not begun receiving minimum distributions.

II and III only. The first statement is incorrect because it is an exception to the 10% penalty for qualified plan distributions, not for IRAs. The second statement applies to IRAs, and third statement is an exception for both qualified plans and IRAs.

All of the following are considered Qualified Benefits for a cafeteria plan EXCEPT: I. Adoption Assistance II. Education Assistance III. Dependent Care Assistance IV. Athletic Facilities

II and IV only.

Retirement Life Expectancy (RLE)

The time period beginning at retirement and ending at death.

Kipton is an executive with BigRock. As part of his compensation, he receives 10,000 shares of restricted stock today worth $20 per share. The shares vest two years from today, at which point the stock is worth $30 per share. The vesting schedule is a 2-year cliff schedule. Kipton holds the stock for an additional 18 months and sells at $45 per share. Which of the following is correct? A. The grant of stock is taxable to Kipton today. B. The value of the shares is taxable to Kipton when the stock vests. C. If Kipton were to make an 83(b) election, he would have converted $30 of the gain from ordinary to capital. D. When Kipton sells the stock for $45 per share, his basis is $30 regardless of whether he files an 83(b) election.

The value of the shares is taxable to Kipton when the stock vests. Choice a is not correct because the stock is forfeitable. Choice c is not correct because it would have converted $10, not $30. Choice d is not correct, because the basis would be different.

Remaining Work Life Expectancy (RWLE)

Period of time remaining to save for future goals. This is typically the time period in which a financial planner has to develop strategies and savings goals because this period begins when they show up at our desk.

Jim, who is age 39, converts a $72,000 Traditional IRA to a Roth IRA in 2019. Jim's adjusted basis in the Traditional IRA is $10,000. He also makes a contribution of $4,000 to a Roth IRA in 2020 for the tax year 2019. If Jim takes a $4,000 distribution from his Roth IRA in 2021, how much total federal income tax, including penalties, is due as a result of the distribution assuming his 2021 federal income tax rate is 22 percent?

$0 Any amount distributed from an individual's Roth IRA is treated as made in the following order (determined as of the end of a taxable year and exhausting each category before moving to the following category): - From regular contributions; - From conversion contributions, on a first-in-first-out basis; and - From earnings. All distributions from all of an individual's Roth IRAs made during a taxable year are aggregated. The 10 percent additional tax under IRC Section 72(t) applies to any distribution from a Roth IRA includible in gross income. The 10 percent additional tax under IRC Section 72(t) also applies to a nonqualified distribution, even if it is not then includible in gross income, to the extent it is allocable to a conversion contribution and if the distribution is made within the 5-taxable-year period beginning with the first day of the individual's taxable year in which the conversion contribution was made.

This year Molly turns 70½. Her life expectancy is 27.4 years and her Roth IRA balance at the end of last year was $500,000. At the end of this year her account balance will be $525,000 and her life expectancy will be 26.5 years. What is her minimum distribution requirement for the current year?

$0 Roth IRA's are exempt from RMD's

Joyce and Melvin have been married for 30 years. In the current year, they received $22,000 of Social Security benefits and had $12,000 of interest income. What portion of the Social Security benefit is taxable?

$0 The lesser of: 50% of $22,000 = $11,000 or 0.5 [$12,000 + 0.5 ($22,000) - $32,000] < 0 Since the answer calculated is less than $0, none of the Social Security benefits received by Joyce and Melvin are taxable.

Natalie is a secretary at JKL Law Firm. JKL provides her with free sodas at her discretion. Natalie estimates that she drinks $20 worth of sodas per month. How much must Natalie include in her annual gross income related to the sodas?

$0 The value of employer-provided deminimis fringe benefits may be excluded from an employee's gross income.

Charles, a single 29 year old, deferred 2% of his salary, or $2,000, into a 401(k) plan sponsored by his employer during 2021. What is the maximum deductible IRA contribution Charles can make during 2021?

$0 Charles cannot make a deductible IRA contribution for the year because he is an active participant in a qualified plan with an AGI of at least $100,000 ($2,000/2%), which exceeds the phase-out limits for a single person of $66,000 - $76,000 for 2021.

Eric, age 53, had the following items of income: Investment returns as a limited partner in a partnership of $1,200. Unemployment compensation of $350. Income from a law practice of $600. Deferred compensation from a former employer of $14,000. Alimony of $750 received from a divorce finalized in 2020. Wages of $1,000. What is the maximum contribution Eric can make to an IRA in 2021?

$1,600 Eric is limited to making an IRA contribution equal to the lesser of $6,000 (2021) (plus the $1,000 catch-up) or his earned income for the year. The following items, totaling $1,600, are considered earned income. Law Practice Income $600 Wages $1,000 The other items are not considered earned income. Individuals must have taxable compensation, such as wages, salaries, commissions, tips, bonuses, or net income from self-employment. Taxable alimony and separate maintenance payments received by an individual are treated as compensation for IRA purposes. Compensation doesn't include earnings and profits from property, such as rental income, interest and dividend income, or any amount received as pension or annuity income, or as deferred compensation. NOTE: Alimony for divorces signed after 12/31/18 is neither taxable by the recipient or deductible by the payor (2017 TCJA).

Going Higher Construction sponsors a 401(k) profit sharing plan. In the current year, Going Higher Construction contributed 25% of each employees' compensation to the profit sharing plan. The ADP of the 401(k) plan for the NHC was 3.5%. If Bob, age 57, earns $100,000 and is a 6% owner, what is the maximum amount that he may defer into the 401(k) plan for this year?

$12,000 Bob is highly compensated because he is more than a 5% owner, so the maximum that he can defer to satisfy the ADP Test requirements is 5.5% (3.5% + 2%) and because he is over 50, he can defer the additional $6,500 (2021) as a catch-up contribution. Bob can defer $5,500 (5.5% × $100,000) and $6,500 (the catch-up) for a total of $12,000.

Hasani died December 31, 2020, at age 50 leaving his wife Jamille and 4 children ages 4, 7, 15 and 17. His PIA is $2,500 per month. Assuming Jamille does not work how much approximately in total per month will she receive for 2021 for herself and kids (ignoring family maximums)?

$13,125 2021 Calculation Jamille is too young to collect as a widow. Widow benefits begin at age 60. Jamille will receive benefit for caring for each of the children under age 16. Each child under age 18 will also receive 75% of the calculated PIA. 3 children (2,500 x 75%) = 5,625 for care 4 children benefits ($2,500 x 75%) = 7,500.

Biks, Inc grants Janie 1 ISO on Jan 1, 20X1. The exercise price is $10. The market price on the exercise date (Jan 1, 20X3) is $25. Janie sold the stock on July 1, 20X3 for $200. What are the tax consequences when Janie sells the stock?

$15 W-2 income; $175 capital gain W-2 income - 25-10 = 15 ST Capital Gain = 200 - 25 = 175

An individual has determined utilizing the annuity method of capital needs analysis that he needs $1,045,656 at the beginning of his retirement to meet his retirement life expectancy goals. If this individual would like to be more conservative in his retirement planning forecast and maintain this capital balance throughout his retirement life expectancy of 32 years, given an expected earnings rate of 6%, and an inflation rate of 3% during the period, how much more would he need to have at the beginning of his retirement?

$162,032

Jim, age 32, earns $65,000 per year. When he retires at age 62 he believes his wage replacement ratio will be 80% and Social Security will pay him $12,000 in today's dollars. How much must Jim save at the end of each year and make the last payment at 62, if he can earn 10% on his investments, inflation is 3% and he expects to live until age 100?

$8,513

Jeff wants to retire in 15 years when he turns 65. Jeff wants to have enough money to replace 75% of his current income less what he expects to receive from Social Security at the beginning of each year. He expects to receive $20,000 per year from Social Security in today's dollars. Jeff is conservative and wants to assume a 6% annual investment rate of return and assumes that inflation will be 4% per year. Based on his family history, Jeff expects that he will live to be 95 years old. If Jeff currently earns $100,000 per year and he expects his raises to equal the inflation rate, approximately how much does he need at retirement to fulfill his retirement goals?

$2,285,172

Kim Cat, age 42, earns $300,000 annually as an employee for CTM, Inc. Her employer sponsors a SIMPLE 401(k) retirement plan and matches all employee contributions made to the plan dollar-for-dollar up to 3% of covered compensation. What is the maximum contribution (employer and employee) that can be made to Kim's SIMPLE 401(k) account in 2021?

$22,200 The maximum total contribution is $22,050. ($13,500 maximum employee contribution for 2021 + $8,700 employer match). The maximum employee contribution for 2021 is $13,500. The employer has chosen to make matching contributions up to 3% of compensation (the SIMPLE maximum). Therefore, the employer can make a contribution of up to $8,700 ($290,000 covered compensation × 3%), if the plan is a SIMPLE 401(k). The SIMPLE 401(k) must follow the qualified plan rules utilizing covered compensation.

Robert Sullivan, age 56, works for Dynex Corporation, and earns $300,000. Dynex Corp. provides a non-elective contribution to its SIMPLE IRA plan. Which one of the following is the maximum amount that could go into Robert's account this year? (The Section 401(a)(17) limit on includible compensation is $290,000 for 2021)

$22,300 The compensation limit applies to SIMPLE IRAs when non-elective contributions are made. $290,000 × 2% = $5,800 + $13,500 (max EE deferral) + $3,000 (age 50+ catch-up) = $22,300. Answer a is correct calculation without the catch-up of $3,000. The incorrect calculation for answers b and d use $300,000 × 2%

Ernest converted his Traditional IRA to a Roth IRA on Dec 15, 2018. He was 35 years of age at the time and had never made a contribution to a Roth IRA. The conversion was in the amount of $60,000 ($10,000 of contributions and $50,000 of earnings). Over the years he has also made $15,000 in contributions. On May 15, 2021 he withdrew the entire account balance of $100,000 to pay for a 1 year trip around the world. Which of the following statements is true?

$25,000 of the distribution will be subject to income tax and $85,000 of the distribution will be subject to the 10% early withdrawal penalty. Roth distributions are tax free if they are made after 5 years and because of 1)Death, 2)Disability, 3) 59.5 years of age, and 4)First time home purchase. He does not meet the five year holding period or one of the exceptions. His distribution does not received tax free treatment. The treatment for a non-qualifying distribution allows the distributions to be made from basis first, then conversions, then earnings. His basis will be tax free. The conversion is also tax free since we paid tax at the time of the conversion on those earnings. The remaining earnings since establishment of the Roth are $25,000 (100,000 - $15,000 in basis - $60,000 in conversions) and will be taxed. The 10% penalty does apply to this distribution since he does not qualify for any of the exceptions to the penalty. The contributions escapes penalty but the conversions and earnings of $85,000 are subject to the 10% early withdrawal penalty. Remember that in order for the conversions to escape the 10% early withdrawal penalty the distribution must occur after a 5 year holding period beginning Jan 1 in the year of conversion or meet one of the 10% early withdrawal exceptions. Summary: $60,000 paid tax at conversion. Subject to penalty $15,000 in contributions no tax, no penalty $25,000 earnings. Taxable and subject to penalty Took out 100,000; $25,000 is taxable, $85,000 subject to penalty

Jared, age 54, earns $300,000 per year and is a participant in his employer's 401(k) plan. Ignoring the ADP Test requirements, what is the maximum amount that Jared can defer under the 401(k) plan in 2021?

$26,000 The general employee elective deferral limitation for 2021 is $19,500, and Jared can defer an additional $6,500 (2021) as a catch-up contribution because he is over 50.

Marilyn Hayward is the sole proprietor and only employee of unincorporated Graphics for Green Promotions. Last year, Marilyn established a profit sharing Keogh plan with a 25% contribution formula. As of the end of this year, Marilyn has $140,000 of Schedule C net earnings. Assume for this problem that her self-employment tax is $19,781. What is the maximum allowable Keogh contribution that Marilyn can make for this year?

$26,022 One-half of self-employment tax is subtracted from the net income subject to self-employment tax that is the $140,000. $140,000 x 92.35% = 129,290 x 15.3% (amount is under the SS wage base) = 19,781.37, then divide by 50% = 9890.68, subtract that from the sched C amount of $140,000-9890.68 and multiply the resulting $130,109.32 by 20%. The contribution of $26,021.88 is 25% of the Koegh base ($130,109 MINUS the Keogh contribution of $26,021 = $104,088 x 25% = 26,022. Incorrect answer A is the maximum 401(k) deferral in 2021. Incorrect answer B is $140,000 × 92.35% × 20%. Incorrect answer D is 25% of $140,000.

Tony Soprano, age 48, earns $460,000 annually as an employee for City Waste Management. His employer sponsors a SIMPLE IRA retirement plan and matches all employee contributions made to the plan dollar-for-dollar up to 3% of compensation. What is the maximum contribution (employer and employee) that can be made to Tony's SIMPLE IRA account in 2021?

$27,000 The maximum total contribution is $27,000 ($13,500 maximum employee contribution for 2021 + 3% employer match × $460,000, but limited to $13,500). An employer cannot match more than was contributed, it is a dollar for dollar match.

Jane is covered by a $90,000 group-term life insurance policy, her daughter is the sole beneficiary. Jane's employer pays the entire premium for the policy; the uniform annual premium is $0.60 per $1,000 per month of coverage. How much, if any, is W-2 taxable income to Jane resulting from the insurance?

$288 $50,000 of group-term life insurance is nontaxable. $90,000 - 50,000 = 40,000 × $0.60 per thousand × 12 = $288 taxable.

You have been hired to analyze the retirement prospects of Tom and Jerri Ruhn. It has been determined they need a retirement capital account of $2,750,000 at retirement which will occur in 30 years. They expect to live in retirement for 35 years. They are anxious to start a savings program to meet this goal. They anticipate an average after-tax rate of return equal to 7%. They are planning on 5% annual inflation. What level of savings put away at the end of each year will provide the Ruhn family with their desired retirement fund?

$29,113 The client has given us the capital account they want at retirement in 30 years. If it said in today's dollars or the equivalent, then you would account for inflation. That information was a distractor in this question. N = 30 I = 7 PV = 0 PMT=29,113 FV = 2,750,000

Kyle is 54 and would like to retire in 11 years. He would like to live the "high" life and would like to generate 90% of his current income. He currently makes $150,000 and expects $24,000 (in today's dollars) in Social Security. Kyle is relatively conservative. He expects to make 8% on his investments, that inflation will be 4% and that he will live until 104. How much does Kyle need at retirement?

$3,554,911

Christine is single, 42, and is an active participant in her employer's qualified retirement plan. Her AGI is $70,000 and she makes the maximum traditional IRA contribution. What is her deductible IRA contribution in 2021?

$3,600 Use the phase out formula. AGI - bottom of the phase out range = ? / range of the phase out (66,000 to 76,000 in 2021) = phase out. [($70,000 - $66,000) / $10,000] = .4 Deduction = $6,000 - (6,000 x 40%) = $3,600

Robin and Robbie, both age 35, are married and filed a joint return for 2021. Robbie earned a salary of $127,000 in 2021 and defers $6,000 to his employer's 401(k) plan. Robbie and Robin earned interest of $15,000 in 2021 from a joint savings account. Robin is not employed, and the couple had no other income. On April 15, 2021, Robbie contributed $6,000 to an IRA for himself and $6,000 to an IRA for Robin. The maximum allowable IRA deduction on the 2021 joint return is:

$6,000 Anyone with earned income can contribute to an IRA, but the ability to deduct the IRA contribution depends on the individual's AGI and whether the individual is an active participant in a qualified plan. Since Robbie has a qualified plan, they cannot deduct the contribution for him due to his income of $136k ($127k - $6k + $15k) which exceeds the AGI phaseout of $105,000 - $125,000 for 2021. Robin, on the other hand, can deduct her contribution because she does not have a qualified plan and their joint income $136k ($121k + $15k) is less than the $198,000 to $208,000 phaseout for the spouse of an active participant. Therefore, Robin's deduction is $6,000. She can use Robbie's earned income as her own for the contribution as she is not employed.

Eric moved from Houston to New Orleans. His expenses for the move included $400 for truck rental, $100 for lodging, and $200 of pre-move house-hunting expenses. If Eric's employer reimbursed him $600, how much of the reimbursement is included in his gross income?

$600 Eric moved from Houston to New Orleans. His expenses for the move included $400 for truck rental, $100 for lodging, and $200 of pre-move house-hunting expenses. If Eric's employer reimbursed him $600, how much of the reimbursement is included in his gross income?

Amy was divorced 2017 and is currently age 55. She received alimony of $51,000 in 2021. In addition, she received $1,800 in earnings from a part-time job. Amy is not covered by a qualified plan. What was the maximum deductible IRA contribution that Amy could have made for 2021?

$7,000 The deductible IRA contribution limit is $6,000 for 2021. The additional catch-up amount, for over age 50, is $1,000 for 2021. Alimony counts as earned income for IRA purposes for divorces finalized prior to 12/31/18 (Pre TCJA). She is not covered by a qualified plan and therefore is not subject to AGI phaseouts. Therefore the total is $7,000 for 2021.

Amy, divorced in 2015 at age 55, received taxable alimony of $52,000 in 2021. In addition, she received $17,000 in earnings from a part-time job. Amy is not covered by a qualified plan. What is the maximum deductible IRA contribution that Amy could have made for 2021?

$7,000 The deductible IRA contribution limit is $6,000 for 2021. The additional catch-up amount, for over age 50, is $1,000 for 2021. Alimony counts as earned income for IRA purposes. She is not covered by a qualified plan and therefore is not subject to AGI phaseouts of 66,000-76,000 for a single tax filer in 2021. Therefore the total is $7,000 for 2021. NOTE: Alimony for divorces signed after 2018 is neither taxable by the recipient or deductible by the payor (2017 TCJA).

A client, age 74, must receive a minimum distribution from his IRA account for this year, which had a value (at the end of the prior year) of $48,000. His spouse, age 73, is the beneficiary of the IRA account. The life expectancy according to IRS tables is 14.1 years. If the client takes a $2,000 distribution on December 31st, what will be the tax penalty if any?

$702 The $48,000 is subject to a 14.1 year life expectancy of the client, thus minimum distribution is $3,404 per year. If the client takes only $2,000, the balance ($1,404) is subject to a 50% excise tax or $702 penalty.

Biks, Inc grants Janie 1 NQSO on Jan 1, 20X1. The exercise price is $10. The market price on the exercise date (Jan 1, 20X3) is $25. Janie sold the stock on July 1, 20X3 for $100. What are the tax consequences when Janie sold the stock?

$75 of short term gain On Jan 1, 20x3 Mkt price - 25 Exercise Price - 10 Gain - 15 - w-2 income On July 1, 20x3 Market Price - 100 Basis - 25 Gain - 75 - short term

A company's defined benefit pension plan utilizes a funding formula that considers years of service and average compensation to determine the pension benefit payable to the plan participants. If Kim is a participant in this defined benefit pension plan and she has 30 years of service with the company and average compensation of $75,000, what is the maximum pension benefit that can be payable to Kim at her retirement?

$75,000 The maximum amount payable from a defined benefit pension plan is the lesser of $230,000 (2021) or 100% of the average of the employee's three highest consecutive years compensation. Because the average of Kim's compensation is $75,000, she would be limited to receiving a pension benefit at her retirement of $75,000.

Donald and Daisy are married and file jointly. They are both age 42, both work, and their combined AGI is $115,000. This year (2021), Donald's profit sharing account earned over $5,000. Neither he nor the company made any contributions and there were no forfeitures. Daisy declined to participate in her company's defined benefit plan because she wants to contribute to and manage her own retirement money. (Her benefit at age 65 under the plan is $240 a month.) How much of their $12,000 IRA contribution can they deduct? Assume that $6,000 is contributed to each account.

$9,000 Daisy is an active participant. She cannot opt out of a defined benefit plan. Use the phase out formula. AGI-bottom of the phase out range=? / range of the phase out (105,000 to 125,000 in 2021) = phase out. $115,000 - $105,000 = $10,000 $10,000/$20,000 = .50 or 50% 50% * $6,000 = $3,000 Donald is not active this year, he will follow the spousal phase out of $198,000 - $208,000. He is eligible for the full $6,000.

Which of the following is/are correct regarding SIMPLE plans? 1. A SIMPLE plan does not require annual testing. 2. A SIMPLE IRA must follow a 3-year cliff vesting schedule if the plan is top-heavy. 3. A 25% early withdrawal penalty may apply to distributions taken within the first two years of participation in a SIMPLE plan. 4. The maximum elective deferral contribution to a SIMPLE 401(k) plan is $19,500 for 2021 and $26,000 for 2021 for an employee who has attained the age of 50.

1 and 3 Statement 1 is correct. Statement 2 is incorrect. A SIMPLE plan is not subject to vesting rules, and contributions are always a 100% vested. Statement 3 is correct. The early withdrawal penalty is 25% for distributions taken within the first two years of participation Statement 4 is incorrect. The maximum deferral to a SIMPLE plan is $13,500 for 2021. Employees who have attained age 50 by the end of the tax year will also be eligible for a catch-up adjustment ($3,000 for 2021).

ABC has an Employee Stock Purchase Plan (ESPP). Which statements regarding an ESPP are correct? 1. The price may be as low as 85% of the stock value. 2. When an employee sells stock at a gain in a qualifying disposition, all of the gain will be capital gain. 3. There is an annual limit of $25,000 per employee.

1 and 3 Statement 2 is incorrect because only the gain in excess of the W-2 income will be capital gain.

Which of the following statements is/are correct regarding TSAs and 457 deferred compensation plans? 1. Both plans require contracts between an employer and an employee. 2. Participation in either a TSA or a 457 plan will cause an individual to be considered an "active participant" for purposes of phasing out the deductibility of Traditional IRA contributions. 3. Both plans allow 10-year forward averaging tax treatment for lump-sum distributions. 4. Both plans must meet minimum distribution requirements that apply to qualified plans.

1 and 4 Statements 1 and 4 are correct. Statement 2 is incorrect because a 457 plan is a deferred compensation arrangement that will not cause a participant to be considered an "active participant." Statement 3 is incorrect because 10-year forward averaging is not permitted from either plan.

Which of the following qualified plans would allocate a higher percentage of the plan's current contributions to a certain class or group of eligible employees? 1. A profit sharing plan that uses permitted disparity 2. An age-based profit sharing plan 3. A defined benefit pension plan 4. A target benefit pension plan

1, 2, 3, and 4 All of the listed plans would allocate a higher percentage of a plans current cost to a certain class of eligible employees.

Which of the following qualified plans require mandatory funding? 1. Defined benefit pension plans 2. 401(k) plans with an employer match organized as a profit sharing plan 3. Cash balance pension plans 4. Money purchase pension plans

1, 3, and 4 401(k) plans do not require mandatory funding. The other three require mandatory funding.

7 Types of Profit Sharing Plans

1. Profit Sharing Plans 2. Stock Bonus Plans 3. Employee Stock Ownership Plans (ESOP/LESOP) 4. 401(k) Plans with either Profit Sharing or Stock Bonus 5. Thrift Plans 6. Age Based Profit Sharing Plans 7. New Comparability Plans

Rick has an 18% nonqualified deferred compensation plan that is funded annually by his employer. Payments are made to a separate trustee of a secular trust who was selected by Rick and his employer. The employer contributions are discontinued at Rick's death, disability, or employment termination. When Rick retires or terminates employment, he will receive the proceeds from the trust. Which of the following is/are correct regarding the deferred compensation plan? 1. The contributions are not currently taxable to Rick because they are subject to a substantial risk of forfeiture. 2. The contributions to the plan are currently subject to payroll taxes. 3. The employer can deduct the contributions to the plan at the time of the contribution.

2 and 3 Because this arrangement is a secular trust, there is no substantial risk of forfeiture. Thus, Statement 1 is false. Because the trust is not subject to the general creditors of the employer, this is straight compensation. Rick must treat the payments as constructively received, and the employer may deduct the payments as compensation immediately. The payments are subject to payroll tax since the compensation is earned.

Marguerite received nonqualified stock options (NQSOs) with an exercise price equal to the FMV at the date of the grant of $22. Marguerite exercises the options 3 years after the grant date when the FMV of the stock was $30. Marguerite then sells the stock 3 years after exercising for $35. Which of the following statements are true? 1. At the date of the grant, Marguerite will have ordinary income of $22. 2. At the date of exercise, Marguerite will have W-2 income of $8. 3. At the date of sale, Marguerite will have long term capital gain of $5. 4. Marguerite's employer will have a deductible expense in relation to this option of $22.

2 and 3 Statements 2 and 3 are correct. Marguerite would not have any taxable income at the date of grant provided the exercise price is equal to the fair market value of the stock. Marguerite's employer would receive a tax deduction equal to the amount of W-2 income Marguerite would be required to recognize, $8 of W-2 income, at the date of exercise. Marguerite's long term capital gain is $5, calculated as the sales price of $35, less the exercise price of $30.

Which of the employee fringe benefits listed below, if provided by the employer, would be included in an employee's gross income? 1. Business periodical subscriptions 2. Season tickets to professional football games 3. Free parking provided near its business (value of $90 per month) 4. The use of an on-premises athletic facilities (value of $180 per month)

2 only Season tickets to professional football games are includible in the gross income of the employee receiving the tickets. All of the other fringe benefits are excludable. Occasional tickets to sporting events would be excludable.

Betty Sue, age 75, is a widow with no close relatives. She is very ill, unable to walk, and confined to a custodial nursing home. Which of the following programs is likely to pay benefits towards the cost of the nursing home? 1. Medicare may pay for up to 100 days of care after a 20-day deductible. 2. Medicaid may pay if the client has income and assets below state-mandated thresholds.

2 only Statement 1 is incorrect because Medicare covers all costs for the first 20 days of skilled nursing home care and cover the next 80 days with a deductible. Students should know from the Insurance course that Medicaid provides for low income persons.

Which of the following statements concerning the use of life insurance as an incidental benefit provided by a qualified retirement plan is (are) correct? 1. The premiums paid for the life insurance policy within the qualified plan are taxable to the participant at the time of payment. 2. Under the 25 percent test, if term insurance or universal life is involved, the aggregate premiums paid for the policy cannot exceed 25 percent of the employer's aggregate contributions to the participant's account. If a whole life policy other than universal life is used, however, the aggregate premiums paid for the whole life policy cannot exceed 50 percent of the employer's aggregate contributions to the participant's account. In either case, the entire value of the life insurance contract must be converted into cash or periodic income, or the policy distributed to the participant, at or before retirement.

2 only Statement 1 is incorrect. The economic value of pure life insurance coverage is taxed annually to the participant. Statement 2 is correct because the 25 percent test is actually a misnomer, for it is really two tests: a 25 percent test and a 50 percent test, depending on which type of life insurance protection is involved.

A distress termination of a qualified retirement plan occurs when: 1. The PBGC initiates a termination because the plan was determined to be unable to pay benefits from the plan. 2. An employer is in financial difficulty and is unable to continue with the plan financially. Generally, this occurs when the company has filed for bankruptcy, either Chapter 7 liquidation or Chapter 11 reorganization. 3. The employer has sufficient assets to pay all benefits vested at the time, but is distressed about it. 4. When the PBGC notifies the employer that it wishes to change the plan due to the increasing unfunded risk.

2 only Statement 2 is the definition of a distress termination. Statement 3 is standard termination. Statement 1 describes an involuntary termination. Statement 4 is simply false.

Patrick and Kevin own Irisha Corporation and plan to retire. They would like to leave their assets to their children; therefore, they transfer 70 percent of the stock to a trust for the benefit of their 10 children pro rata. Patrick and Kevin then plan to sell the remaining Irisha shares to a qualified ESOP plan. Which of the following is correct? 1. The stock transfer to the ESOP is not a 50 percent transfer and therefore will not qualify for non-recognition of capital gains. 2. Any transfer to an ESOP of less than 50 percent ownership may be subject to a minority discount on valuation.

2 only There must have been a sale of at least 30% (not 50%) to the ESOP to qualify for non-recognition of capital gain treatment. In addition, any transfer that is less than 50% of the stock of the corporation might be subject to a minority discount on valuation.

A business valued at $3,000,000 has 3 partners. Each of the 3 partners buys a $500,000 life insurance policy on each of the other partners. Which of the following is true? 1. This is an example of an entity purchase plan. 2. This is an example of a cross purchase plan. 3. The policies are under funded.

2 only This is a cross-purchase life insurance plan. Each person has a one-third interest. Therefore, when the first partner dies, the other two partners will each need to pay $500,000 for a total of $1,000,000 (1/3 of $3,000,000). Thus, the policies are not underfunded.

Which of the following vesting schedules may a top-heavy qualified profit sharing plan use? A. 2 to 7 year graduated. B. 5 year cliff. C. 2 to 4 year graduated. D. 4 to 8 year graduated.

2 to 4 year graduated. As a result of the PPA 2006, qualified profit sharing plans must use a vesting schedule that provides participants with vested benefits at least as rapidly as either a 2 to 6 year graduated vesting schedule or a 3-year cliff vesting schedule. This requirement applies without regard to whether the profit sharing plan is a top-heavy plan. Employers can be more generous, never less. Options a, b, and d all vest less rapidly than the required schedule.

Which of the following statements is/are correct regarding SEP contributions made by an employer? 1. Contributions are subject to FICA and FUTA. 2. Contributions are currently excludable from employee-participant's gross income. 3. Contributions are capped at $19,500 for 2021.

2. Statement 2 is the only correct response. Statements 1 and 3 are incorrect. Employer contributions to a SEP are not subject to FICA and FUTA. The 401(k) elective deferral limit and the SARSEP deductible limits are $19,500 for 2021. The SEP limit is 25% of covered compensation up to $58,000 for 2021. Note: The maximum compensation that may be taken into account in 2021 for purposes of SEP contributions is $290,000. Therefore, the maximum amount that can be contributed to a SEP in 2021 is $58,000 (25% × $290,000, limited to $58,000).

Which of the following employees is a key employee for 2021? 1. Matt, an officer of the company, who earns $100,000 per year and owns 2% of the company. 2. Missy, who earns $13,000 per year and owns 5% of the company. 3. Tara, an officer of the company who earns $200,000. 4. Julie, a 10% owner of the company who earns $4,000 per year as a secretary.

3 and 4 Only Tara and Julie are considered key employees. Missy is not a greater than 5% owner, she has 5%, which does not meet the criteria. A key employee is anyone who is any one or more of the following (1) a greater than five percent owner, or (2) a greater than one percent owner with compensation in excess of $150,000, or (3) an officer with compensation in excess of $185,000 (2021).

Generally, which of the following are contributory plans? A. 401(k) and money purchase pension plans. B. 401(k) and thrift plans. C. Thrift plans and ESOPs. D. Money purchase pension plans and profit sharing plans.

401(k) and thrift plans. Employers generally contribute to Money Purchase Pension Plans, ESOPs, and Profit Sharing Plans. Employees contribute (thus contributory plans) to 401(k)s and Thrift Plans.

Marcus has been employed by GCD Enterprises for 15 years, and currently earns $60,000 per year. Marcus saves $15,000 per year. He plans to pay off his home at retirement and live debt free. He currently spends $12,000 per year on his mortgage. What do you expect Marcus' wage replacement ratio to be based on the above information?

47.35%

Hasani died December 31, 2020, at age 50 leaving his wife Jamille and 4 children ages 4, 7, 15 and 17. His PIA is $2,500 per month. How many checks will his wife receive (assuming direct deposits) in 2021?

5 1 for her and 4 to her for the benefit of each child

Packlite company has a defined benefit plan with 200 non-excludable employees (40 HC and 160 NHC). They are unsure if they are meeting all of their testing requirements. What is the minimum number of total employees that must be covered on a daily basis to conform with the 50/40 test?

50 The 50/40 rule requires that defined benefit plans cover the lesser of 50 employees or 40% of all eligible employees. Here 40% would be 80, so 50 is less than 80. This would be the absolute minimum number of covered employees.

Packlite company has a defined benefit plan with 200 nonexcludable employees (40 HC and 160 NHC). They are unsure if they are meeting all of their testing requirements. What is the minimum number of total employees that must be covered on a daily basis to conform with the requirements set forth in the IRC?

50 The 50/40 rule requires that defined benefit plans cover the lesser of 50 employees or 40% of all eligible employees. Here 40% would be 80, so 50 is less than 80. This would be the absolute minimum number of covered employees.

Jon, age 48, earns $65,000 gross per year from his employer. Jon saves $15,000 per year for retirement and pays $12,000 per year for his home mortgage. Given this information and considering that Jon will have eliminated his mortgage debt before retirement, what is Jon's expected wage replacement ratio during retirement?

50.81%

Marie, the sole shareholder in Marie's Pastries, is contemplating establishing a qualified plan. Marie, a long-time widow, has always treated the employees like her family and the company has experienced very low turnover. She would like to use the retirement plan to assist her in transferring ownership interest to the employees as she is ready to retire. She has a strong preference for avoiding and deferring taxes. She is opposed to mandatory funding and indifferent to integration. Which plan would be appropriate for Marie given the corporation's employee census is as follows: (Focus on the bold items to get to the right plan.)

An ESOP would be the most appropriate plan to meet Marie's objectives. The stock bonus plan would allow Marie to transfer stock, but would not assist her immediately in her retirement plans. The defined benefit and money purchase pension plan would require mandatory funding. The ESOP would provide her with tax benefits and a diversified portfolio because of her age.

Wage Replacement Ratio (WRR)

An estimate of the percent of income needed at retirement compared to earnings prior to retirement.

The following statements concerning retirement plan service requirements for qualified plans are correct EXCEPT: A. The term "year of service" refers to an employee who has worked at least 1,000 hours during the initial 12-month period after being employed. B. According to the Internal Revenue Code, if an employee hired on October 5, 20X1 has worked at least 1,000 hours or more by October 4, 20X2, he has acquired a year of service the day after he worked his 1,000th hour. C. An employer has the option of increasing the one-year of service requirement to 2 years of service. D. Once an employee attains the service requirement of the plan, the employer cannot make the employee wait more than an additional six months to enter the plan.

B. Option B is incorrect because the employee would NOT acquire a year of service the day after he worked his 1,000th hour, but after twelve months AND 1,000 hours

Which of the following is true regarding Sensitivity Analysis? A. It is a mathematical tool that utilizes a probabilistic distribution of returns and their effects on an individual's retirement plan. B. It is the changing of variable assumptions to determine the effects to the retirement plan. C. It is a test used to determine if a plan passes the ADP test. D. It is a tool to determine if a Roth IRA distribution is taxable.

B. Sensitivity Analsys is the changing of variable assumptions to determine the effects to the retirement plan. This is typically done by rotating a variable toward increased risk. For example, if inflation assumption is 3%, what would happen to the plan if it actually turns out to be 3.5% or 4%. Monte Carlo Analysis is a mathematical tool that utilizes a probabilistic distribution of returns and their effects on an individual's retirement plan.

June and Bud, both 40 years old, are not covered by a qualified retirement plan. Bud, trying to maximize their IRA deduction, put $12,000 into an IRA with June as the beneficiary on December 15 of the current year. What best describes the result of this transaction? A. June and Bud receive a tax deduction for the entire $12,000 because both spouses are eligible to contribute $6,000 to the IRA. B. Bud receives a tax deduction for $6,000 and a 6% penalty for over-contribution on the other $6,000. C. Next year Bud will receive a $6,000 deduction, in addition to the $6,000 deduction for this year. D. Bud receives a tax deduction for $6,000 and is considered to have made a non-deductible contribution of the other $6,000.

B. This question indicates an IRA in only Bud's name. Maximum contribution is $6,000 (2021) plus any applicable catch-up provisions. Amounts contributed over that level are considered excess contributions and subject to a 6% penalty until taken out. The 6% penalty could have been avoided if the excess contribution was withdrawn prior to the original filing deadline without extension.

Which of the following concerning the Social Security system is correct? A. Workers entitled to retirement benefits can currently take early retirement benefits as early as age 60. B. A worker who takes early retirement benefits will receive a reduced benefit because he or she will receive more monthly benefit payments as payments commence earlier than if the worker had waited and retired at full retirement age. C. Family members of an individual who is eligible for retirement or disability benefits include a spouse if the spouse is at least 60 years old or under 62 but caring for a child under age 16. D. Generally, individuals who are over the age of 62 and receive Social Security benefits automatically qualify for Medicare benefits.

B. Workers entitled to retirement benefits can currently take early retirement benefits beginning at age 62. Family members of an individual who is eligible for retirement or disability benefits include a spouse if the spouse is at least 62 years old or under 62 but caring for a child under age 16. Generally, individuals who are over age 65 and receive Social Security benefits automatically qualify for Medicare.

Which of the following statements concerning choosing the most appropriate type of vesting schedule for a qualified plan --restrictive vs. generous--is (are) correct? 1. Two advantages of choosing a restrictive vesting schedule are (1) to reduce costs attributable to employee turnover and (2) to help retain employees. 2. Three advantages of choosing a liberal vesting schedule in which there is immediate and full vesting are (1) to foster employee morale (2) keep the plan competitive in attracting employees, and (3) to meet the designs of the small employer who desires few encumbrances to participation for the "employee family."

Both 1 and 2

XYZ has a noncontributory qualified profit sharing plan with 310 employees in total, 180 who are nonexcludable (40 HC and 140 NHC). The plan covers 72 NHC and 29 HC. The NHC receive an average of 4.5% benefit and the HC receive 6.5%. Which of the following statements is (are) correct? 1. The XYZ company plan meets the ratio percentage test. 2. The XYZ company plan fails the average benefits test. 3. The plan must and does meet the ADP test.

Both 1 and 2 The plan does not have to meet the ADP test because it is a noncontributory plan. The plan meets the ratio percentage test and fails the average benefits test. Safe Harbor = 72 ÷ 140 = 51% = Fail Ratio % = (72 ÷ 140) ÷ (29 ÷ 40) = 70.9% = Pass Average Benefit = 4.5 ÷ 6.5 = 69.2% = Fail

Which of the following statements concerning choosing the most appropriate type of vesting schedule for a qualified plan --restrictive vs. generous--is (are) correct? I. Two advantages of choosing a restrictive vesting schedule are (1) to reduce costs attributable to employee turnover and (2) to help retain employees. II. Three advantages of choosing a liberal vesting schedule in which there is immediate and full vesting are (1) to foster employee morale (2) keep the plan competitive in attracting employees, and (3) to meet the designs of the small employer who desires few encumbrances to participation for the "employee family."

Both I and II. Both statements are correct.

All of the following statements concerning cash balance pension plans are correct EXCEPT: A. The cash balance plan is generally motivated by two factors: selecting a benefit design that employees can more easily understand, and as a cost saving measure. B. The cash balance plan is a defined benefit plan. C. The cash balance plan has no guaranteed annual earnings to participants. D. The cash balance plan is subject to minimum funding requirements.

C. A basic component of a cash balance plan is the guaranteed minimum investment return.

Which of the following clauses in a 401(k) plan can assist the plan in meeting the requirements of the ADP test? A. Attestation clause. B. No-Contest clause. C. Automatic election clause. D. Deferral plan clause.

C. A negative election clause can assist a 401(k) plan in meeting the ADP test because it automatically deems that an employee defers a specific amount unless he elects out of the automatic deferral amount. Options A and D do not exist and Option B is a clause commonly found in a will.

Robbie is the owner of SS Automotive and he would like to establish a qualified pension plan. Robbie would like most of the plan's current contributions to be allocated to his account. He does not want to permit loans and he does not want SS Automotive to bear the investment risk of the plan's assets. Robbie is 32 and earns $700,000 per year. His employees are 25, 29, and 48 and they each earn $25,000 per year. Which of the following qualified pension plans would you recommend that Robbie establish?

C. Because Robbie does not want SS Automotive to bear the investment risk of the plan assets, the money purchase pension plan or the target benefit plan would be the available options to fulfill his requirements. The target benefit plan would not fulfill Robbie's desires because as a percentage of compensation, older employees receive a greater contribution in a target benefit plan and one of the employees is older than Robbie. In such a case, the older employee would receive a greater (as a percentage of compensation) contribution to the plan.

Wilber receives incentive stock options (ISOs) with an exercise price equal to the FMV at the date of the grant of $15. Wilber exercises these options 3 years from the date of the grant when the FMV of the stock is $35. Wilber then sells the stock 3 years after exercising for $45. Which of the following statements is (are) true? A. At the date of grant, Wilber will have ordinary income equal to $15. B. At the date of exercise, Wilber will have W-2 income of $20. C. At the date of sale, Wilber will have long-term capital gain of $30. D. Wilber's employer will have an income tax deduction related to the exercise of the option by Wilber.

C. Choice a is not correct as there is no income at the date of grant because the strike price equals the FMV. Choice b is not correct as there is no regular tax for ISOs. Choice d is not correct because the employer will not have an income tax deduction. Grant 15 Exercise @35 (AMT adjustment of $20) Sale $45 $10 LTCG from gain (exercise to sale) $20 LTCG from exercise Total $30 LTCG More information can be found in the inforgraphics posted in Blackboard, under helpful documents and infographics, scroll down for NQSO and ISO.

All of the following are reasons that an employer might favor a nonqualified plan over a qualified retirement plan except: A. There is more design flexibility with a nonqualified plan. B. A nonqualified plan typically has lower administrative costs. C. Nonqualified plans typically allow the employer an immediate income tax deduction. D. Employers can generally exclude rank-and-file employees from a nonqualified plan.

C. Nonqualified plans do not allow the employer to take an income tax deduction until the employee recognizes the income. All of the other statements are correct.

Which of the following people is definitely considered highly compensated for 2021? A. Renee, a one percent owner who earns $80,000 per year. B. Hannah, who earned $120,000 last year and is the 40th highest paid employee of 100 employees. C. Conrad, a 20 percent owner who earns $40,000 per year. D. Daniel, a five percent owner who earns $45,000 per year.

C. Only Conrad would be considered highly compensated because he is greater than a five percent owner. An individual is deemed highly compensated if he is either a greater than five percent owner, or has earnings in excess of $130,000 (2021) and is in the top 20 percent, as ranked by salary of all employees (if the employer chooses to elect the 20% rule. This rule would help limit who was considered a HCE).

Joe Liner works at a company that is considering options regarding its future legacy payments and it needs to find current tax deductions. One option the company is considering is funding a VEBA this year. Joe is uneasy but open to the idea because he has heard that more benefits may be funded in the VEBA. Which of the following are permitted under a VEBA? I. Life, sickness and accident benefits II. Retirement benefits III. Severance and supplemental unemployment IV. Job training V. Commuter benefits

C. Retirement benefits and commuter benefits cannot be included in a VEBA.

Which of the following is not a qualified retirement plan? A. ESOP. B. 401(k) plan. C. 403(b) plan. D. Target benefit plan.

C. 403(b) plan. A 403(b) plan is a tax-advantaged plan, not a qualified plan. All of the others are qualified plans.

Match the following statement with the type of retirement plan which it most completely describes: "A qualified plan which allows employee elective deferrals of 100% of includible salary and has a mandatory employer match" is... A. A Profit-sharing plan. B. A Money purchase plan. C. A SIMPLE 401k. D. A Defined benefit plan.

C. A SIMPLE 401(k). Profit sharing plans "A" are not contributory. Answers "B" and "D" do not permit employee elective deferrals.

Robbie is the owner of SS Automotive and he would like to establish a qualified pension plan. Robbie would like most of the plan's current contributions to be allocated to his account. He does not want to permit loans and he does not want SS Automotive to bear the investment risk of the plan's assets. Robbie is 32 and earns $700,000 per year. His employees are 25, 29, and 48 and they each earn $25,000 per year. Which of the following qualified pension plans would you recommend that Robbie establish? A. Target benefit pension plan B. Cash balance pension plan C. Money purchase pension plan D. Defined benefit pension plan using permitted disparity

C. Money purchase pension plan Because Robbie does not want SS Automotive to bear the investment risk of the plan assets, the money purchase pension plan or the target benefit plan would be the available options to fulfill his requirements. The target benefit plan would not fulfill Robbie's desires because as a percentage of compensation, older employees receive a greater contribution in a target benefit plan and one of the employees is older than Robbie. In such a case, the older employee would receive a greater (as a percentage of compensation) contribution to the plan.

Each of the following are requirements imposed by law on qualified tax-advantaged retirement plans EXCEPT: A. Plan documentation B. Employee vesting C. Selective employee participation D. Employee communications

C. Selective employee participation Broad employee participation, as opposed to selective participation, is a requirement of a tax-advantaged retirement plan. All of the others are requirements for "qualified" plans.

Workaholism

Characterized by a fear of not having enough money and as a result focus on their career at the expense of personal relationships. They often justify their focus on work and avoidance of family and friends. It is a compulsive disorder that is often associated with anxiety or depression.

Hoarding

Characterized by accumulating and being unable to discard possessions that most people would consider worthless. The accumulation of these types of worthless possessions often creates clutter that negatively impacts living and working spaces. Hoarding disorder can also be associated with money such that one is unwilling to spend money even after becoming successful.

Financial Enabling

Characterized by continuing to provide financial support to someone and depriving them of developing their own financial acumen and responsibility. It is often seen with a parent child relationship but can be with siblings or others that one may care for. It is often associated with feelings of guilt or the need to feel wanted / important.

Which of the following accurately describes a 403(b) plan? A. A 403(b) plan is a noncontributory qualified profit sharing plan. B. Because of catch-up provisions, the investment risk of the assets within a 403(b) plan is borne equally by the plan sponsor and the participant. C. A participant's contributions will generally vest according to a 3 to 7 year graduated vesting schedule, however, a 5-year cliff vesting schedule may be used. D. 403(b) plan assets can be invested indirectly in stocks and bonds through annuities or mutual funds.

D. Answer D is a correct statement accurately describing a 403(b) plan. Answer A is incorrect as a 403(b) plan is an employee deferral plan and is not a qualified plan. Answer B is incorrect as the investment risk is borne by the employee in all cases. Answer C is incorrect as an employee's contributions within a 403(b) plan is always 100% vested.

All of the following statements concerning the Social Security system are correct except: A. If a worker receives retirement benefits based on his or her own earnings record, the worker's retirement benefits will continue whether married or divorced. B. Widows and widowers, whether divorced or not, will continue to receive survivors benefits upon remarriage if the widow or widower is age 60 or older. C. By providing the name of a country or countries to be visited and the expected departure and return dates, the Social Security Administration will send special reporting instructions to the beneficiaries and arrange for delivery of checks while abroad. D. A special one-time payment of $1,050 may be made to a deceased worker's spouse or minor children upon death.

D. A special one-time payment of $255 may be made to a deceased worker's spouse or minor children upon death.

Which of the following would reduce the amount needed to be saved on an annual basis in order to accumulate sufficient retirement assets? A. Accelerated inflation. B. Expansion of current lifestyle. C. Excess returns on investments over projections. D. Increased life expectancy due to medical advances.

Excess returns on investments over projections.

A SEP is not a qualified plan and is not subject to all of the qualified plan rules. However, it is subject to many of the same rules. Which of the following are true statements? I. SEPs and qualified plans have the same funding deadlines (due date of return plus extensions). II. The contribution limit for SEPs and qualified plans (defined contribution) is $58,000 for the year 2021. III. SEPs and qualified plans have the same ERISA protection from creditors. IV. SEPs and qualified plans have different nondiscriminatory and top-heavy rules.

I and II SEPs and qualified plans can be funded as late as the due date of the return plus extensions. The maximum contribution for an individual to a SEP is $58,000 for 2021 ($290,000 maximum compensation × 25%, limited to $58,000). Thus, statements 1 and 2 are correct. Qualified plans are protected under ERISA. IRAs and SEPs do not share this protection. Both types of plans have the same nondiscriminatory and top-heavy rules.

Which of the following fringe benefits would be included in taxable income? I. Season tickets to the professional baseball team ($2,000 value per year). II. Access to a local gym, not owned by the employer ($800 value per year). III. One-time use of the company dump truck to deliver sand to an employee's residence ($500 value). IV. A crystal plaque worth $150 as an achievement award.

I and II only. A. Season tickets are taxable. B. To be deductible/excludable, the gym must be on the employer's property or under the employer's control. C. De minimus use is not taxable. D. Awards cannot be cash, but a plaque is fine.

Investment portfolio risk is generally borne by the participant/employee in all of the listed qualified plans, EXCEPT: I. Defined benefit pension plans. II. Cash balance pension plans. III. 401(k) plans. IV. Profit sharing plans.

I and II only. In defined benefit and cash balance pension plans, the employer bears the investment risk.

A SEP is not a qualified plan and is not subject to all of the qualified plan rules. However, it is subject to many of the same rules. Which of the following are true statements? I. SEPs and qualified plans have the same funding deadlines. II. The contribution limit for SEPs and qualified plans (defined contribution) is $58,000 for the year 2021. III. SEPs and qualified plans have the same ERISA protection from creditors. IV. SEPs and qualified plans have different nondiscriminatory and top-heavy rules.

I and II only. SEPs and qualified plans can be funded as late as the due date of the tax return plus extensions. The maximum contribution for an individual to a SEP is $58,000 for 2021 ($290,000 maximum compensation × 25%, limited to $58,000). Thus, statements I and II are correct. Qualified plans are protected under ERISA. IRAs and SEPs do not share this protection. Both types of plans have the same nondiscriminatory and top-heavy rules.

Which of the following are permitted investments in a 403(b) TSA (TDA) plan? I. An annuity contract from an insurance company. II. An international gold stock mutual fund. III. A self-directed brokerage account consisting solely of U.S. stocks, bonds and mutual funds.

I and II only. TSA (TDA) funds can only invest in annuity contracts (Statement I) and mutual funds (Statement II). No self-directed brokerage accounts are permitted.

Ricky receives stock options for 12,000 shares of XYZ Corporation with an exercise price of $10 when the stock is trading on the national exchange for $10 per share. The XYZ company plan is an Incentive Stock Option Plan. Which of the following statements are true regarding the options? I. Ricky will be required to hold any ISOs for more than a year after exercise and more than two years from the grant date to have long-term capital gains. II. 2,000 of the options are considered NQSOs.

I and II. To the extent the fair market value of the stock for which the ISO is exercisable for the first time during any calendar year exceeds $100,000, the excess is treated as a nonstatutory stock option; therefore, 2,000 of the options are NQSOs.

Which of the following individuals are "key employees" as defined by the Internal Revenue Code? I. A more-than-5% owner of the employer business. II. An employee who received compensation of more than $120,000 from the employer. III. An officer of the employer who received compensation of more than $190,000. IV. A 1% owner of the employer business having annual compensation from the employer of more than $55,000.

I and III only. A key employee is an individual who (1) owns more than 5% of the business, (2) is an officer with compensation greater than $185,000 (2021), or (3) owns at least 1% of the business and has compensation greater than $150,000. I. and III. are defined key employees. II. defines a highly compensated employee, not a key employee. IV. should state that compensation was more than $150,000.

Which of the following is/are correct regarding SIMPLE plans? I. A SIMPLE plan does not require annual testing. II. A SIMPLE IRA must follow a 3-year cliff vesting schedule if the plan is top-heavy. III. A 25% early withdrawal penalty may apply to distributions taken within the first two years of participation in a SIMPLE plan. IV. The maximum elective deferral contribution to a SIMPLE 401(k) plan is $19,500 for 2021 and $26,000 for 2021 for an employee who has attained the age of 50 or older.

I and III only. Statement I is correct. Statement II is incorrect. A SIMPLE plan is not subject to vesting rules, and contributions are always a 100% vested. Statement 3 is correct. The early withdrawal penalty is 25% for distributions taken within the first two years of participation Statement IV is incorrect. The maximum deferral to a SIMPLE plan is $13,500 for 2021. Employees who have attained age 50 by the end of the tax year will also be eligible for a catch-up adjustment ($3,000 for 2021).

The early distribution penalty of 10 percent does not apply to qualified plan distributions: I. Made after attainment of the age of 55 and separation from service. II. Made for the purpose of paying qualified higher education costs. III. Paid to a designated beneficiary after the death of the account owner who had not begun receiving minimum distributions.

I and III only. Statement II is an exception for distributions from IRAs, not qualified plans. Statements I and III are exceptions to the 10% penalty for qualified plan distributions.

ABC has an Employee Stock Purchase Plan (ESPP). Which statements regarding an ESPP are correct? I. The price may be as low as 85% of the stock value. II. When an employee sells stock at a gain in a qualifying disposition, all of the gain will be capital gain. III. There is an annual limit of $25,000 per employee.

I and III only. Statement II is incorrect because only the gain in excess of the W-2 income will be capital gain.

Which of the following statements is/are correct regarding TSAs and 457 deferred compensation plans? I. Both plans require contracts between an employer and an employee. II. Participation in either a TSA or a 457 plan will cause an individual to be considered an "active participant" for purposes of phasing out the deductibility of Traditional IRA contributions. III. Both plans allow 10-year forward averaging tax treatment for lump-sum distributions. IV. Both plans must meet minimum distribution requirements that apply to qualified plans.

I and IV only. Statements I and IV are correct. Statement II is incorrect because a 457 plan is a deferred compensation arrangement that will not cause a participant to be considered an "active participant." Statement III is incorrect because 10-year forward averaging is not permitted from either plan.

Generally, which of the following plans favor older entrants? I. Defined Benefit Plan II. Cash Balance Pension Plan III. 401 (k) Plan with a Profit Sharing Plan IV. Profit Sharing Plan

I only.

Which of the following types of 457 plans permit employees to defer recognition of income without a risk of forfeiture? I. Public 457(b) plans. II. 457(f) plans. III. Private 457(b) plans.

I only. 457(f) plans and private 457(b) plans must be subject to a substantial risk of forfeiture.

Which of the following statements concerning rabbi trusts is (are) CORRECT? I. A rabbi trust is a trust established and sometimes funded by the employer that is subject to the claims of the employer's creditors, but any funds in the trust cannot generally be used by or revert back to the employer. II. A rabbi trust calls for an irrevocable contribution from the employer to finance benefits promised under a nonqualified plan, and funds held within the trust cannot be reached by the employer's creditors. III. A rabbi trust may not be held off-shore as a result of the American Jobs Creation Act of 2004. IV. The American Jobs Creation Act of 2004 prohibits "springing irrevocability" for a rabbi trust if there is a change of control or ownership.

I only. II describes a secular trust. III is incorrect because off-shore Rabbi trusts may still create and hold assets but there is no tax benefit for doing so. Realistically, these are no longer created because of the loss in preferential tax treatment. However, any off-shore Rabbi trust previously created is grandfathered so as long as there are no material changes to the plan it may maintain the pre-AJCA '04 treatment (the preferential tax deferral). IV is wrong because AJCA 2004 does allow springing irrevocability in these circumstances, but not for bankruptcy.

Which of the following statements is/are correct regarding 403(b) plans? I. 403(b)s are eligible for rollover treatment to IRAs, qualified plans, and other 403(b)s. II. Investments in stocks, bonds, and money markets are available. III. Assets in a 403(b) plan are always 100% vested.

I only. Statement I is correct as 403(b) plans are permitted to be rolled over to IRAs, qualified plans, or other 403(b)s. Statement II is incorrect because 403(b) investments are limited to mutual funds and insurance annuities. Statement III is incorrect because, while employee elective deferrals are always 100% vested, employer contributions may be subject to a vesting schedule—typically ERISA limited vesting schedules (2-6 graded or 3-year cliff).

RCM Incorporated sponsors a qualified plan that requires employees to meet one year of service and to be 21 years old before being considered eligible to enter the plan. Which of the following employees are not eligible? I. Donald, age 18, who has worked full-time with the company for 3 years. II. Rachel, age 22, who has worked full-time with the company for 6 months. III. Randy, age 62, who has worked 500 hours per year for the past 6 years. IV. Theodore, age 35, who has worked full-time with the company for 10 years.

I, II and III only. RCM cannot exclude anyone who has attained age 21 and has completed one year of service with the company with 1,000 hours during that year.

Which of the following statements are correct regarding a retirement plan? I. Employees don't start earning benefits until they become plan participants (except in defined benefit plans, which may count prior service). II. Since turnover is generally highest for employees in their first few years of employment and for younger employees, it makes sense from an administrative standpoint to delay eligibility. III. A tax-exempt educational institution may delay eligibility to age 26 under certain circumstances.

I, II and III. All statements are typical potential qualified plan provisions to delay eligibility of employees to participate in a retirement plan.

Which of the following plans permit employers to match employee elective deferral contributions or make nonelective contributions? I. 457(b) II. 401(k) III. 403(b)

I, II and III. All three plans permit employer matching and nonelective contributions. The 457 employer contribution goes against the annual limit, whereas employer contributions do not go against the annual limit for 401(k) and 403(b) plans.

Which of the following statements is/are correct regarding the early distribution 10 percent penalty tax from a qualified plan? I. Retirement at age 55 or older exempts the distributions from the early withdrawal penalty tax. II. Distributions used to pay medical expenses in excess of 7.5% of AGI (2021) for a tax filer who itemizes are exempt from the early withdrawal penalty. III. Distributions that are part of a series of equal periodic payments paid over the life or life expectancy of the participant are exempt from the early withdrawal penalty.

I, II and III. TCDTR Act of 2020 set medical expense deduction to 7.5% of AGI permanently.

Which of the following plans require mandatory funding? I. Defined Benefit Plan II. Cash Balance Pension Plan III. ESOP IV. Target Benefit

I, II and IV only. An ESOP does not have mandatory funding requirements.

Persons that are deemed to be a "disqualified person" include: I. Plan sponsor. II. Plan Fiduciary. III. Officers of plan sponsor. IV. Family members of owners.

I, II, III and IV.

Which of the following statements accurately reflects the tax consequences of contributions to and distributions from HSAs? I. Contributions by the employee are deductible for AGI. II. Distributions for medical expenses are tax-free. III. Contributions by the employer are excludable from income by the employee. IV. Earnings within the HSA are non-taxable.

I, II, III and IV.

Qualified retirement plans that permit the employer unlimited investment in sponsor company stock are: A. 401(k) plans. B. Stock bonus plans. C. Profit sharing plans. D. ESOPs.

I, II, III and IV. All of the listed plans permit 100% stock in the plans. The 401(k) plan is organized as a profit sharing or stock bonus plan.

Which of the following qualified plans would permit allocating a higher percentage of the plan's current contributions to a certain class or group of eligible employees? I. A profit sharing plan with a 401(k) provision and, therefore, calculating permitted disparity. II. An age-based profit sharing plan. III. A defined benefit pension plan. IV. A target benefit pension plan.

I, II, III and IV. All of the listed plans would permit allocating a higher percentage of a plan's current cost to a certain class of eligible employees.

What benefits are available to the survivors of a deceased worker who was currently insured but not fully insured at death? I. Lump sum death benefit of $255. II. Mother or father's spousal benefit for caring for a qualifying child under age 16. III. Income benefits to a child under age18. IV. Survivor benefit to spouse (assume not remarried) at age FRA.

I, II, and III. There are no survivor benefits to a surviving spouse with no qualifying child.

Which of the following statements accurately reflect the characteristics of a Section 457 plan? I. Benefits taken as periodic payments are treated as ordinary income for taxation. II. Lump-sum distributions are eligible for 5-year and/or 10-year averaging. III. Deferred amounts are subject to Social Security and Medicare taxes at the later of: performance of services or employee becomes vested in the benefits. IV. Income tax withholding is not required until funds are actually received, not constructively received. V. Cannot exceed the smaller of $19,500 or 100% includible compensation in 2021.

I, III and V only. There are no special tax advantages provided for 457 plans distributed in a lump-sum. Income tax withholding is required once the benefits are constructively received, even if not actually received.

Qualified retirement plans have which of the following characteristics? I. Employees with one year of service and attained age 21 must be participants in the plan. II. Fund earnings are usually not taxed until distributions are received by the employee. III. All lump-sum distributions are eligible for five-year forward averaging tax treatment. IV. Employer contributions to the plan are deductible in the year they are made (or deemed made), subject to IRC Section 415 limits.

II and IV only. Maximum waiting period for qualified plans is two years (except for SEPs [employer sponsored tax advantaged plan] which can have a 3-year waiting period). No lump sum distributions are eligible for 5-year averaging after December 31, 1999.

Which of the legal requirements apply to defined benefit pension plans? I. Each participant must have a separate account to hold assets. II. An actuary is needed to calculate the minimum funding level. III. Retirement benefits can be adjusted based on sponsor profits. IV. The benefits in most traditional defined benefit plans are protected, within certain limitations, by federal insurance provided through the Pension Benefit Guaranty Corporation (PBGC).

II and IV only. Statement "I" applies only to defined contribution plans. Statement "III" describes a profit-sharing plan.

A SEP-IRA is a form of defined contribution plan (although not a qualified plan). Which of the following apply to BOTH the SEP-IRA and a traditional defined contribution plan? I. Employer deductions limited to 15% of covered payroll. II. Requires a definite, written, non-discriminatory contribution allocation formula. III. Contributions cannot discriminate in favor of highly compensated employees. IV. Employer contributions subject to Medicare and Social Security taxes. V. Affiliated service group rules apply. VI. Top-heavy rules do NOT apply. VII. Permissible disparity or integration is NOT allowed.

II, III and V only. Defined contribution plans have an employer deductibility limit of 25% of covered payroll. All defined contribution plans must have a written allocation formula so assets can be distributed in the mandated individual accounts. Employer contributions must bear uniform resemblance to compensation and cannot discriminate in favor of highly compensated. Employer contributions are not subject to any payroll related taxes. Top-heavy rules do apply to both. Both plans can integrate with Social Security (sometimes called permissible disparity). (Note: 5305-SEP does not allow permissible disparity.)

Which of the following employees is a key employee for 2021? I. Matt, an officer of the company, who earns $100,000 per year and owns 2% of the company. II. Missy, who earns $13,000 per year and owns 5% of the company. III. Tara, an officer of the company who earns $195,000. IV. Julie, a 10% owner of the company who earns $4,000 per year as a secretary.

III and IV only. Only Tara and Julie are considered key employees. A key employee is anyone who is any one or more of the following: (1) A greater than five percent owner, or (2) a greater than one percent owner with compensation in excess of $150,000, or (3) an officer with compensation in excess of $185,000 (2021).

Kent Reeder, age 52, works as the administrator and curator at the Museum of Antique Manuscripts, a not-for-profit organization in Metropolitan Center. He has worked there 18 years and began contributing to the 403(b) plan 12 years ago but skipped contributing last year. He earns $85,000 a year. He has asked you to maximize his contribution. Which of the following is/are TRUE? I. He may contribute $19,500 plus $6,500 for age 50+ catch-up, plus $3,000 long service catch-up. II. He may not contribute to the long-service catch-up this year due to omitting a contribution last year. III. He may contribute $19,500 plus $6,500 age 50+ catch-up. IV. He may not participate in both the long service catch-up and the age 50+ catch-up in the same year. V. He is not eligible for the long service catch-up.

III and V He is not eligible for the long service catch-up because the museum is not a HER (health, education, religious) organization. The maximum contribution limits for 2021 are $19,500 plus the age 50+ catch-up of $6,500.

Kent Reeder, age 52, works as the administrator and curator at the Museum of Antique Manuscripts, a not-for-profit organization in Metropolitan Center. He has worked there 18 years and began contributing to the 403(b) plan 12 years ago but skipped contributing last year. He earns $85,000 a year. He has asked you to maximize his contribution. Which of the following is/are TRUE? I. He may contribute $19,500 plus $6,500 for age 50+ catch-up, plus $3,000 long service catch-up. II. He may not contribute to the long-service catch-up this year due to omitting a contribution last year. III. He may contribute $19,500 plus $6,500 age 50+ catch-up. IV. He may not participate in both the long service catch-up and the age 50+ catchup the same year. V. He is not eligible for the long service catch-up.

III and V only. He is not eligible for the long service catch-up because the museum is not a Health, Education, Religious (HER) organization. The maximum contribution limits for 2021 are $19,500 plus the age 50+ catch-up of $6,500.

Carolyn Smart wanted to volunteer full-time and decided to retire from Lotsa Cash Corporation at the age 57, after 15 years of service. She requested a total distribution of her account in the Lotsa Cash Corporation's profit sharing plan and received a check, made payable to her. Her account balance was $60,000 on her final day of employment. Which of the following statements describe the consequences of this distribution? I. Eligible for 10 year forward averaging. II. Subject to 10% penalty. III. Eligible for Rollover. IV. Subject to mandatory 20% withholding. V. Exempt from the 10% early withdrawal penalty.

III, IV and V only She must be born by 1/1/1936 in order to use 10 year forward averaging. She is not subject to the 10% penalty due to separation of service after age 55. She is eligible for a rollover. She is subject to the 20% withhold since the check went to her (indirect rollover) She is exempt from the 10% penalty, due to separation of services after age 55.

Inflation calculation

N = 10, 20, 30, 40 or 50 I = 3 PV = ? PMT = 0 FV = 100,000

Tom, age 39, is an employee of Star, Inc., which has a profit sharing plan with a CODA feature. His total account balance is $412,000, $82,000 of which represents employee elective deferrals and earnings on those deferrals. The balance is profit sharing contributions made by the employer and earnings on those contributions. Tom is 100 percent vested. Which of the following statements is/are correct? I. Tom may take a loan from the plan, but the maximum loan is $41,000 and the normal repayment period will be 5 years. II. If Tom takes a distribution (plan permitting) to pay health care premiums (no coverage by employer) he will be subject to income tax, but not the 10% penalty.

Neither 1 nor 2 Statement 1 is incorrect because he can take a loan equal to one-half of his total account balance up to $50,000. Statement 2 is incorrect because the exemption from the 10% penalty only applies to IRAs and only to the unemployed.

Christine has been the owner of Chris' Antique Dolls for the past 15 years. She decided to establish a retirement plan for her corporation. She wants to make all initial contributions to the plan using company stock and she may integrate with social security. Which of the following would be the best qualified plan for them to consider adopting?

Profit sharing plan A profit sharing plan will allow a stock contribution and integration with social security. A stock bonus plan would also be an appropriate, but it's not one of the choices.

Brisco, age 51, is the Executive VP of sales at Doggie Daycare (DD). His base salary is $300,000 with a potential bonus of 50%. Brisco is a participant in his employer's 401(k) plan and always defers the maximum amount. The DD 401(k) plan has the following features and characteristics: Includes a Roth account that is not a safe harbor plan, but has a 50% match up to 4%. The ADP for the NHC is 4.5%. The plan has $3 million in assets that are managed by two asset management firms. DD also sponsors a defined benefit plan that provides a benefit based on years of service and final salary. The DD DB plan provides for 1.5 percent per year of service for the first 20 years and 2 percent for years above 20, up to a max of 35 years. On the weekends, Brisco paints murals. His entity, Wall Works LLC (WW), is a single owned LLC taxed as a disregarded entity. Brisco would like to establish a retirement plan for the income that he earns in WW. He expects to earn $60,000 ever year in WW and wants to know what the best retirement plan is for his business. Which plan would you recommend for him?

SEP Neither a SIMPLE nor a 401(k) plan will work because he is already deferring $18,850 (6.5% (4.5% + 2%) times $290,000) to his 401(k) plan. Therefore, the choices are a defined benefit plan (expensive) or a SEP, which is extremely easy to set up and one that he can contribute around $11,000 to annually. Instructor note: He is currently contributing 6.5% of salary capped at 290,000 for 2021, which is 18,850. The most he can contribute to multiple 401k plans is limited to 19,500. A 401k is rather expensive with ongoing filing for him to contribute $650 (19,500 -18,850). The most he can do is a match as the employer, the 401k did not state it had a Profit sharing component (nor did it state it was a solo 401k). A SEP is much cheaper to set up and has no annual report requirements like the 401k. He was be able to contribute as an employer to the SEP, which will be based on 20% of net income, around the $11,000 amount (without doing the full self-employment calculation).

A qualified plan which allows employee elective deferrals of 100% of includible salary and has a mandatory employer match

SIMPLE 401(k)

Mary Anne has AGI of $1,000,000 (which is all comprised of earned income). She is single and age 55. She is not an active participant in her employer's qualified plan. Which of the following statements best describes her options? A. She can contribute to a Traditional IRA and deduct her contribution. B. She can contribute to a Traditional IRA but not deduct her contribution. C. She can contribute to a Roth IRA. D. She cannot contribute to a Traditional IRA or Roth IRA.

Solution: The correct answer is A. She can contribute and deduct her contribution to a Traditional IRA since she is not an active participant and therefore not subject to an AGI limitation. She is unable to contribute to a Roth IRA because she is above the AGI limitation of $125,000 - $140,000 (2021).

Gerry is 70 on April 1, 2021 and has an account balance of $423,598 as of the end of last year. If Gerry takes a $15,000 distribution in December 2021, what is the amount of the minimum distribution tax penalty?

The correct answer is $0. SECURE Act 2019 changed Required Minimum Distributions to age 72 as a start date for those that reach 70 1/2 after 12/31/19. Gerry will not need to begin RMDs for 2 years. Any distributions prior to that will not count towards the RMD. 6% is the penalty for over contribution. The insufficient withdrawal penalty is 50% of the amount not taken $7,730 is the amount calculated under the pre-SECURE Act rules

Which of the following is not true regarding profit sharing plans? A. The plan is established and maintained by the individual employee. B. Allows employees to derive benefit from profits of the company. C. Profit sharing plans cannot discriminate in favor of officers and shareholders. D. Profit sharing plans provide a definite predetermined formula for allocating the contributions made to the plan among the participants and for distributing the funds accumulated under the plan.

The correct answer is A. Option A is not true regarding profit sharing plans. A profit sharing plan is established and maintained by the employer. The remaining options are true statements.

Which of the following is not a requirement for the owner of corporate stock who sells to an ESOP to qualify for the nonrecognition of gain treatment? A. The ESOP must own at least 55% of the corporation's stock immediately after the sale. B. The owner must reinvest the proceeds from the sale into qualified replacement securities within 12 months after the sale. C. The ESOP may not sell the stock within three years of the transaction unless the corporation is sold. D. The owner must not receive any allocation of the stock through the ESOP.

The correct answer is A. The ESOP must own at least 30% of the corporation's stock immediately after the sale. All of the other statements are true.

Kevin is age 62 and collecting Social Security benefits. In order to begin receiving Medicare Part A benefits, he must: A. Do nothing, coverage starts immediately at age 65. B. File a separate application for Medicare upon his 65th birthday. C. Do nothing, coverage starts immediately upon receiving retirement benefits, regardless of age. D. File a separate application for Medicare upon his 67th birthday.

The correct answer is A. Coverage starts automatically. If you receive retirement benefits early, there's no need to file a separate application.

Which of the following statements accurately states the tax consequences for group health insurance premiums paid by an employer? A. Non-deductible for employer and excluded from taxable income for employee. B. Deductible for employer and excluded from taxable income for employee. C. Non-deductible for employer and included in taxable income for employee. D. Deductible for employer and included in taxable income for employee.

The correct answer is B.

All of the following statements regarding Social Security are correct except: A. The worker who takes early retirement benefits will receive a reduced benefit. B. Workers entitled to retirement benefits can currently take early retirement benefits as early as age 59 ½. C. To qualify for retirement benefits, a worker must be fully insured, which means that a worker has earned a certain number of quarters of coverage under the Social Security system. D. Earning a designated amount of money, regardless of when it was earned during the year, will credit the worker with a quarter of coverage for that year.

The correct answer is B. As early as age 62 not 59½.

Andrea died this year (2021) at the age 77, leaving behind a qualified plan worth $200,000. Andrea began taking minimum distributions from the account after attaining age 72 and correctly reported the minimum distributions on her federal income tax returns. Before her death, Andrea named her granddaughter, Reese age 22, as the designated beneficiary of the account. Now that Andrea has died, Reese has come to you for advice with respect to the account. Which of the following is correct? A. Reese must distribute the entire account balance within five years of Andrea's death. B. Reese must distribute the entire account balance within ten years of Andrea's death. C. In the year following Andrea's death, Reese must begin taking distributions over Andrea's remaining single-life expectancy. D. Reese can roll the account over to her own name, treat the account as her own and name a new beneficiary.

The correct answer is B. SECURE Act 2019 changed distribution rules for beneficiaries of account owners that died after 12/31/19. Whether the account owner died before RBD (Required Begin Date) or after, the distribution rules are now the same. All Designated Beneficiaries must withdraw the account balance within 10 years of the owner's death. Eligible Designated Beneficiaries may distribute over their life expectancy in the year following owner's death. Eligible Designated Beneficiaries are: •Surviving spouse for the employee or IRA owner •Child of employee or IRA owner who has not reached majority •At age of majority becomes a designated beneficiary •Chronically ill individual •Any other individual who is not more than ten years younger than the employee or IRA owner Non-Designated Beneficiaries (no listed Beneficiary) rules are pending clarification from the IRS but we believe must be distributed within 5 years of the account owner's death. The new rules were not clear if the difference for before or after RBD were still applicable. Reese is more than 10 years younger than Andrea, which makes her a Designated Beneficiary.

Maximum Performance, Inc.'s defined contribution plan has been determined to be top heavy. Which one of the following statements is NOT a requirement that applies to the plan? A. The employer must contribute a minimum of 3% of compensation or the contribution rate of the key employees (whichever is lower) per year to non-excludable, non-key employees for each year that the plan is top heavy. B. If the employer contribution to key employees is 2%, then the employer contribution to non-excludable, non-key employees must be 2%. C. The plan must use a vesting schedule that does not exceed either a 2-year cliff or 6-year graded vesting schedule. D. The plan must fully vest after three years of service if the vesting at two years is zero.

The correct answer is C.

Which of the following statements regarding an age-based profit sharing plan is correct? A. An age-based profit sharing plan provides a greater benefit to those plan participants whose earnings exceed the Social Security wage base and who are over fifty years old. B. An age-based profit sharing plan only provides a benefit to those plan participants whose age is within 10 years of the age of the owner of the plan sponsor. C. An age-based profit sharing plan provides greater benefits to older plan participants. D. Younger plan participants in an age-based profit sharing plan usually receive the majority of the profit sharing plan allocation.

The correct answer is C. An age-based profit sharing plan provides a greater benefit to older plan participants as the allocation of the plan contribution is based upon the age of the participants. Options A and B are false statements without any merit. Option D is incorrect because OLDER plan participants in an age-based profit sharing plan usually receive the majority of the profit sharing plan allocation.

All of the following statements regarding Social Security are correct except: A. Many private insurance companies sell Medicare supplemental insurance polices. B. Medicare supplemental insurance policies help pay Medicare's coinsurance amounts and deductibles, as well as other out-of-pocket expenses for health care. C. If a worker applies for retirement or survivor's benefits before his or her 65th birthday, he or she must file a separate application for Medicare. D. Even if an individual continues to work after turning 65, he or she should sign up for Part A of Medicare.

The correct answer is C. If a worker applies for retirement or survivor's benefits before his or her 65th birthday, there is no need to file a separate application for Medicare

Which of the following cannot be held in an IRA account as an investment? A. A U.S. gold coin B. Option contracts (calls) C. Variable life insurance D. Municipal bonds

The correct answer is C. Life insurance is not permitted in IRA accounts. All of the other choices are permissible.

Robin began taking required minimum distributions from her profit sharing plan several years ago. Robin died after suffering a heart attack on January 2, 2021. She had named her twin sister Johanna as beneficiary of her profit sharing plan. Which of the following statements is false? A. Johanna may take a full distribution of the profit sharing plan's assets in the year of Robin's death. B. After Johanna's death, her named beneficiary will need to distribute the balance of the account within 10 years of Johanna's death. C. Robin's sister must take a distribution of the profit sharing plan account balance by the end of the fifth year after the year of her death. D. The required minimum distribution can be taken over Johanna's life, the year following Robin's death.

The correct answer is C. SECURE Act 2019 changed the distribution rules following the account owner's death. The new rules do not differentiate between death before or after RMDs start. Johanna is an Eligible Designated Beneficiary and will be able to distribute the account over her life expectancy. Any balance upon Johanna's death is subject to the 10 year rule. Johanna could elect to take the distributions faster than her life expectancy if she wishes. She will not be subject to distribution within 5 years

Brandon's employer gave him restricted stock worth $100,000 (5,000 shares at $20 per share) on January 1st of the current year. Five years later, when all restrictions are lifted, the stock will be trading at $40 per share. Which of the following accurately describes the tax consequences for Brandon in 5 years? A. $200,000 LT capital gain. B. $100,000 W-2 income and 100,000 LT capital gain. C. $200,000 W-2 income. D. $100,000 LT capital gain.

The correct answer is C. The stock is taxable when it vests and is treated as W-2 income. Since there is a five year vesting schedule, it will be entirely W-2 income a that time. Brandon could have filed an 83(b) election upon the granting of the stock and converted the $20 of appreciation per share into capital gains.

All of the following events qualify for 36 months of COBRA coverage EXCEPT: A. The group health plan terminates. B. Death of the employee. C. Employee reached Medicare age. D. Normal termination.

The correct answer is D.

The Third Party Administrator (TPA) of the Flying Trapeze Manufacturing Incorporated's Defined Contribution Plan has just informed you, its administrator, that the plan is top heavy. Which one of the following statements is NOT a requirement that applies to your plan? A. Employees must be 100% vested in the plan after three years of service if the vesting at two years is zero. B. If the employer contribution to key employees is 2%, then the employer contribution to non-excludable, non-key employees may be reduced to no lower than 2%. C. Flying Trapeze, Inc. must contribute a minimum of 3% of compensation or the contribution rate of the key employees, if it is lower, to non-excludable, non-key employees for each year that the plan is top heavy. D. The plan's vesting schedule must be 100% vested upon participation.

The correct answer is D. A is correct because the plan must have no longer than a 3 year cliff or 2-6 year graded vesting. If year 2 is zero, it must be a cliff vesting and year 3 must be 100%.

All of the following statements concerning Social Security beneficiaries are correct except: A. Monthly benefits can be paid to a disabled insured worker under age 65. B. Benefits can be paid to the divorced spouse of a retired or disabled worker entitled to benefits if age 62 or over and married to the worker for at least 10 years and 1 day. C. Benefits can be paid to the surviving spouse (including a surviving divorced spouse) of a deceased insured worker if the widow(er) is age 60 or over. D. Benefits can be paid to dependent parents of a deceased insured worker at age 59 or over.

The correct answer is D. Benefits can be paid to the dependent parents of a deceased worker at age 62 or over.

Josh recently died on January 5, 2021 at the age of 63, leaving a qualified plan account with a balance of $1,000,000. Josh was married to Kay, age 53, who is the designated beneficiary of the qualified plan. Which of the following is correct? A. Kay must distribute the entire account balance within five years of Josh's death. B. Kay must begin taking distributions over Josh's remaining single-life expectancy. C. Any distribution from the plan to Kay will be subject to a 10 percent early withdrawal penalty until she is 59½. D. Kay can receive annual distributions over her remaining single-life expectancy.

The correct answer is D. Kay can receive distributions over her remaining single-life expectancy. Kay qualifies as an eligible designated beneficiary as she is 10 years younger, not more than 10 years younger. Answer A is incorrect. She is not required to distribute the entire account within 5 years. Answer B is incorrect. Kay can wait (not must) until Josh would have been 72 to begin taking distributions over her recalculated life expectancy. Answer C is incorrect. The distribution will not be subject to the early withdrawal penalty because the distributions were on account of death.

Endorsement Split-dollar life insurance is: A. An insurance arrangement in which the employee pays the cost of the premium and the employee names the employer as the beneficiary. B. An insurance arrangement in which the employer and the employee share the cost of the life insurance on the employee and the portion of the premium that is paid by the employer is the value of the term life portion of the policy. C. An insurance arrangement in which the employee pays the majority of the premium while the employer names the beneficiary. D. An insurance arrangement in which the employer is the owner of the policy and is also the beneficiary to the extent of the premiums paid by the employer.

The correct answer is D. The employer is the owner of an endorsement split-dollar policy and will be repaid the total of the premiums it has paid for the insurance.

Which of the following is a characteristic of a defined benefit retirement plan: A. The plan estimates the benefit an employee receives at retirement. B. The plan has predictable costs as compared to defined contribution plans. C. The plan assigns the risk of pre-retirement inflation, investment performance, and adequacy of retirement income to the employee. D. The law specifies the maximum allowable benefit payable from the plan is equal to the lesser of 100% of salary or $230,000 (2021) per year currently.

The correct answer is D. Option D describes characteristics of a defined benefit plan. DB plans have three benefit calculations covered in your textbook; unit benefit, flat dollar and flat percent. Only unit benefit uses 100% of average final compensation (or average highest 3 years). All use the max benefit of $230,000. A is incorrect; corrected: The plan SPECIFIES the benefit an employee receives at retirement. B is incorrect; corrected: The plan has LESS predictable costs as compared to defined contribution plans. C is incorrect; corrected: Defined benefit plans assign the risk of pre-retirement inflation, investment performance, and adequacy of retirement income to the EMPLOYER.

Which of the following accurately describes a 403(b) plan? 1. A 403(b) plan is a noncontributory qualified profit sharing plan. 2. Because of catch-up provisions, the investment risk of the assets within a 403(b) plan is borne equally by the plan sponsor not the participant. 3. A participant's contributions within a 403(b) plan will generally vest according to a 3 to 7 year graduated vesting schedule, however, a 5-year cliff vesting schedule may be used. 4. 403(b) plan assets can be invested indirectly in stocks and bonds through annuities or mutual funds.

The correct answer is D. Option D is a correct statement accurately describing a 403(b) plan. Option A is incorrect as a 403(b) plan is an employee deferral plan and is not a qualified plan. Option B is incorrect as the investment risk is borne by the employee in all cases. Option C is incorrect as an employee's contribution within a 403(b) plan is always 100% vested.

Medical Trials Inc. has a cafeteria plan. Full-time employees are permitted to select any combination of the benefits listed below, but the total value received by each employee must be $6,500 a year or less. 1. Group medical and hospitalization insurance for employee only, $3,600 a year. 2. Group medical and hospitalization insurance for employee's spouse and dependents, $1,200 additional a year. 3. Child-care payments, actual cost not to exceed $5,000. 4. Cash required to bring the total of benefits and cash to $6,500. 5. Universal variable life insurance $1,000. Which of the following statements is true? (All employees are full time) A. James chooses to receive $6,500 cash because his wife's employer provides medical benefits for him. James has $2,900 of taxable income ($6,500 - $3,600). B. Matt chooses 1, 2, 5, and $700 cash. He must include $700 in taxable income. C. Randy chooses 1 and 2 and $1,700 in child care. He must include the $1,700 in gross income. D. Robin chooses 1 and 2 and $1,700 cash. Robin must include $1,700 in taxable income.

The correct answer is D. Option D is correct because cash must be included in income. Option A is incorrect because the entire cash distribution will be taxable. Option B is incorrect because the universal variable life insurance premiums of $1,000 cannot be excluded from Matt's gross income. Option C is incorrect because child care payments are excludable benefits.

Meb, the owner of Meb's Hardware, is considering establishing a stock bonus plan. She recently talked with her financial planner, Don T. Know. Don T. Know never studied when he took his certificate program, therefore he gave Meb incorrect information about stock bonus plans. Which of the following statements given to Meb was correct? A. Meb can require the employees to be age 21 and employed for three years before becoming eligible for the stock bonus plan. B. When the employee's of Meb's Hardware receive distributions of stock from the stock bonus plan, they will receive capital gain treatment on the distribution equal to the value of the stock as contributed by Meb's Hardware. C. A valuation of the stock of Meb's Hardware is required when the stock bonus plan is established, but subsequent valuations are unnecessary. D. Meb can establish a stock bonus plan for the previous year anytime before the due date (plus extensions) of Meb's Hardware's tax return.

The correct answer is D. SECURE Act 2019 - allows establishment of a Qualified Plan by filing date of tax return with extensions, following the standards set by the SEP plan. When the employees of Meb's Hardware receive distributions of stock from the plan, the value of the distribution equal to the value at the date Meb contributed the stock will be ordinary income. The appreciation will be long-term capital gain. Meb must have the stock valued at the date the plan is established and at the date of any distributions.

Your Uncle Ben began receiving required minimum distributions from his IRA in several years ago and has died leaving a balance in his IRA. Uncle Ben is your Dad's youngest brother, but is 13 years your senior. He has named you as beneficiary (non-spouse beneficiary). Which of the following identifies your minimum distribution rule? A. Distributions may be made over your life expectancy (fixed) beginning by December 31st in the year of his death. B. Distributions may be made only over a five-year period. C. Distributions must be made over Uncle Ben's life expectancy (recalculated) with the first distribution made by December 31st of the year following death. D. Distribution of the full account must be made within 10 years of Uncle Ben's death. E. Distributions must be made in a lump sum.

The correct answer is D. SECURE Act 2019 changed the distribution rules following an account owner's death after 12/31/19. Only an Eligible Designated Beneficiary will be able to distribute based on their life expectancy. All Designated Beneficiaries will be required to distribute the full account balance within 10 years of the account owner's death. Answer A is incorrect because the distributions to the beneficiary are based on the beneficiary's life expectancy beginning in the year following death. Do point out that the distribution in the year of death is still based on the owner's life (pre-SECURE Act). Answer B is incorrect because the five-year rule is for non-designated beneficiaries (no listed beneficiary). Answer C is incorrect because a non-spouse beneficiary would not receive distributions based on the owner's life expectancy nor would a post-death non-spouse distribution be recalculated. (pre-SECURE Act) Answer E is incorrect because a lump sum is NOT the only choice. You can always receive money faster than required.

Which of the following is true regarding QDROs? A. The court determines how the retirement plan will satisfy the QDRO (i.e., split accounts, separate interest). B. In order for a QDRO to be valid, the order must be filed on Form 2932-QDRO provided by ERISA. C. All QDRO distributions are charged a 10% early withdrawal penalty. D. A QDRO distribution is not considered a taxable distribution if the distribution is deposited into the recipient's IRA or qualified plan.

The correct answer is D. The plan document, not the court, determines how the QDRO will be satisfied. No particular form is required for a QDRO, although some specific information is required. Form 2932-QDRO is not a real form.

Which of the following is true regarding QDROs? A. The court determines how the retirement plan will satisfy the QDRO (i.e. split accounts, separate interest). B. In order for a QDRO to be valid, the order must be filed on Form 2932-QDRO provided by ERISA. C. All QDRO distributions are charged a 10% early withdrawal penalty. D. A QDRO distribution is not considered a taxable distribution if the distribution is deposited into the recipient's IRA or qualified plan.

The correct answer is D. The plan document, not the court, determines how the QDRO will be satisfied. No particular form is required for a QDRO, although some specific information is required. Form 2932-QDRO is not a real form. QDRO distributions may be subject to the 10% early withdrawal penalty if the distribution is not deposited into the recipient's IRA or qualified plan.

Which of the following will be subject to a 10% early withdrawal penalty? A. Sylvia, age 56, retired from Marshall Corporation. She takes a $125,000 distribution from the Marshall Corporation Defined Contribution Retirement Plan to pay for living expenses until she is eligible for Social Security. B. Terry quits Shoe Shine Company at age 48. He begins taking equal distributions over his life expectancy from his qualified plan after separating from service. The annual distribution is $2,000. C. Kevin leaves Hedwig University at age 50. He takes a $1,000,000 distribution from his defined contribution pension plan. Six weeks after receiving the $800,000 check (net of 20% withholding), Kevin deposits $1,000,000 into a new IRA account. D. Edward, age 40, takes a $40,000 distribution from his profit-sharing plan to pay for his son's college tuition.

The correct answer is D. There is no provision for a distribution without penalty under these circumstances. Edward is only 40 and education withdrawals are allowed from IRAs, but not from qua

Which of the following benefits provided by an employer to its employees is currently taxable to the employee? A. Employees of the DEF Department Store are allowed a 15% discount on store merchandise. DEF's normal gross profit percentage is 20%. B. On a space-available basis, undergraduate tuition is waived by Private University for the dependent children of employees (value of $15,000 per semester). C. Fly Airline allows its employees to fly free when there are open seats available on a flight (average value of $200). D. Incidental personal use of a company car.

The correct answer is D. Personal use of a company car is a taxable fringe benefit. All of the other employer fringe benefits listed may be excluded from the employee's gross income.

Work Life Expectancy (WLE)

This is the complete time period available to work and earn in order to save for retirement.

As retirement life expectancy increases, there is an increased need to finance the retirement life expectancy and a shortened work life expectancy in which to save and accumulate assets.

True

Wage Replacement Ratio (WRR) The Top-Down Approach

Uses percentages and common sense. Uses more estimates and is more efficient for younger individuals because of the uncertainty of future income and expenditures.

Angelo's Bakery has 105 employees. 90 of the employees are nonexcludable and 15 of those are highly compensated (75 are nonhighly compensated). The company's qualified profit sharing plan benefits 8 of the highly compensated employees and 40 of the nonhighly compensated employees. Does the profit sharing plan sponsored by Angelo's Bakery meet the coverage test?

Yes, the plan meets the ratio percentage test. The plan meets the ratio percentage test. The percentage of NHC employees covered by the plan is 53.33% and the percentage of HC employees covered by the plan is 53.33%. The ratio percentage of the NHC employees covered by the plan compared to the ratio percentage of the HC employees covered by the plan is 100% (53.33% / 53.33%) which is greater than the ratio requirement of at least 70%.


Ensembles d'études connexes

Fundamentals of Business Law Chapter 4

View Set

Chapter 3: "MANAGING AND USING DATA"

View Set

AC 351 Chapter 9 Flexible Budgets and Performance Analysis

View Set

Physical Science Chapter 1 Density, Mass, and Volume 8th Grade

View Set

Cell Organelles and Function with Labels

View Set